Moc zbioru

Algebra zbiorów. Relacje, funkcje, iloczyny kartezjańskie... Nieskończoność, liczby kardynalne... Aksjomatyka.
kt26420
Użytkownik
Użytkownik
Posty: 99
Rejestracja: 21 sty 2021, o 16:29
Płeć: Kobieta
wiek: 21
Podziękował: 40 razy

Moc zbioru

Post autor: kt26420 »

Chcę pokazać, że moc zbioru funkcji nierosnących z \(\displaystyle{ \NN}\) w \(\displaystyle{ \NN}\) jest alef 0.
Umiem to pokazać od dolu ograniczenie, ale nie wiem jak zrobić ograniczenie z góry.
Czy mógłby ktoś pomóc?
Ostatnio zmieniony 5 gru 2021, o 17:40 przez Jan Kraszewski, łącznie zmieniany 1 raz.
Powód: Poprawa wiadomości.
Awatar użytkownika
Janusz Tracz
Użytkownik
Użytkownik
Posty: 4060
Rejestracja: 13 sie 2016, o 15:01
Płeć: Mężczyzna
Lokalizacja: hrubielowo
Podziękował: 79 razy
Pomógł: 1391 razy

Re: Moc zbioru

Post autor: Janusz Tracz »

Każda z takich funkcji jest od pewnego miejsca stała więc
\(\displaystyle{ \left\{ f\in\NN^{\NN}:f \text{ jest nierosnąca} \right\}= \bigcup_{c\in\NN}^{}\left\{ f\in\NN^{\NN}:f \text{ jest nierosnąca oraz od pewnego miejsca równa }c\right\}. }\)
Ponad to \(\displaystyle{ (\forall c)\left| \left\{ f\in\NN^{\NN}:f \text{ jest nierosnąca oraz od pewnego miejsca równa }c\right\}\right|=\aleph_0 }\) bo zbiór ciągów od pewnego miejsca stały jest przeliczalny. Zatem \(\displaystyle{ \left\{ f\in\NN^{\NN}:f \text{ jest nierosnąca} \right\}}\) jest przeliczalny jako przeliczalna suma zbiorów przeliczalnych.

Dodano po 8 minutach 15 sekundach:
A to, że ciągów od pewnego miejsca stale równych \(\displaystyle{ c}\) jest \(\displaystyle{ \aleph_0}\) to wynika z zanurzenia
\(\displaystyle{ \left\langle \alpha _1, \alpha _2,\dots, \alpha_{\text{ostatnie miejsce przed ccc...}}\right\rangle \mapsto \prod^{\text{tyle ile trzeba}}_{p-\text{pierwsze}} p_i^{ \alpha _i}}\)
Jakub Gurak
Użytkownik
Użytkownik
Posty: 1392
Rejestracja: 20 lip 2012, o 21:19
Płeć: Mężczyzna
Lokalizacja: Rzeszów
Podziękował: 60 razy
Pomógł: 83 razy

Re: Moc zbioru

Post autor: Jakub Gurak »

To, że zbiór wszystkich ciągów od \(\displaystyle{ n}\)- tego miejsca równych \(\displaystyle{ m\in\NN}\) jest przeliczalny, wynika po prostu stąd, że od \(\displaystyle{ n}\)-tego miejsca każdy taki ciąg jest wyznaczony jednoznacznie, więc możliwości tworzenia takich różnych ciągów są tylko na \(\displaystyle{ n}\)- pierwszych pozycjach. Ponieważ są to ciągi liczb naturalnych, więc chyba łatwo pokazać (dla \(\displaystyle{ n\in\NN_+}\), dla \(\displaystyle{ n=0}\), dla dowolnego ustalonego \(\displaystyle{ m\in\NN,}\) jest dokładnie jeden taki ciąg), chyba łatwo pokazać przez indukcję, że zbiór takich ciągów, oznaczmy go jako \(\displaystyle{ X_{n,m}}\) spełnia: \(\displaystyle{ X_{n,m}\sim \NN^{n}\sim \NN.}\)
Jakub Gurak
Użytkownik
Użytkownik
Posty: 1392
Rejestracja: 20 lip 2012, o 21:19
Płeć: Mężczyzna
Lokalizacja: Rzeszów
Podziękował: 60 razy
Pomógł: 83 razy

Re: Moc zbioru

Post autor: Jakub Gurak »

Również, gdyż udowodniłem ten fakt przedwczoraj, mamy taki fakt, że wszystkich funkcji na zbiorze liczb całkowitych ujemnych wraz z zerem i o wartościach też w nim, wszystkich takich funkcji nierosnących jest przeliczalnie wiele. W ukrytej treści poniżej przedstawiam dowód tego faktu.
Ukryta treść:    
Również, udowodniłem przedwczoraj, że wszystkich funkcji na zbiorze liczb całkowitych ujemnych wraz z zerem, wszystkich funkcji silnie rosnących jest continuum. W ukrytej treści poniżej przedstawiam dowód tego ciekawego faktu.
Ukryta treść:    
I również wszystkich funkcji na zbiorze liczb całkowitych ujemnych wraz z zerem, wszystkich takich funkcji słabo rosnących jest dokładnie continuum.

Tzn. jeśli \(\displaystyle{ X=\ZZ_- \cup \left\{ 0\right\}}\), a rodzina \(\displaystyle{ \mathbb{B}}\) jest dana jako:

\(\displaystyle{ \mathbb{B}=\left\{ f:X \rightarrow X\Bigl| \ \ f \hbox{ jest słabo rosnąca}\right\}}\),

to ta rodzina wszystkich funkcji na zbiorze liczb całkowitych ujemnych wraz z zerem, rodzina wszystkich funkcji słabo rosnących ma moc continuum.


PROSTY DOWÓD TEGO FAKTU:

Rozważmy rodzinę funkcji \(\displaystyle{ \mathbb{A}}\), daną jako:

\(\displaystyle{ \mathbb{A}=\left\{ f:X \rightarrow X\Bigl| \ f \hbox{ jest silnie rosnąca}\right\}.}\)

Wiemy już, na mocy faktu udowodnionego powyżej przed chwilą, że ta rodzina ma moc continuum.

Wykażemy, że \(\displaystyle{ \mathbb{B}\supset \mathbb{A}.}\)

Niech \(\displaystyle{ f\in \mathbb{A}}\). Wtedy \(\displaystyle{ f:X \rightarrow X,}\) i funkcja \(\displaystyle{ f}\) jest silnie rosnąca. Ponieważ funkcja \(\displaystyle{ f}\) jest silnie rosnąca, więc w szczególności \(\displaystyle{ f}\) jest słabo rosnąca, a zatem \(\displaystyle{ f\in\mathbb{B}}\), a zatem \(\displaystyle{ \mathbb{A}\subset \mathbb{B}.}\)

A zatem \(\displaystyle{ \left| \mathbb{B}\right| \ge \left|\mathbb{A}\right| =\left| \RR\right|.}\)

Z drugiej jednak strony, możemy rodzinę \(\displaystyle{ \mathbb{B}}\) ograniczyć przez: \(\displaystyle{ \mathbb{B}\subset X^X}\), i ponieważ \(\displaystyle{ \ZZ_- \cup \left\{ 0\right\} =X\sim \NN}\), więc jest to zbiór równoliczny z \(\displaystyle{ \NN ^{\NN}}\), który to zbiór jest równoliczny ze zbiorem \(\displaystyle{ \RR}\). A zatem \(\displaystyle{ \left| \mathbb{B}\right| \le \left| \RR\right|}\). Mamy \(\displaystyle{ \left|\mathbb{B}\right| \ge \left| \RR\right|}\), a zatem, na podstawie twierdzenia Cantora-Bernsteina: \(\displaystyle{ \mathbb{B}\sim \RR. \square}\) :lol:


A tutaj, w ostatnim moim poście badałem, m.in., funkcje rosnące, malejące na zbiorze liczb całkowitych. Zainteresowani mogą kliknąć i zobaczyć. 8-)
ODPOWIEDZ